Edits
This commit is contained in:
parent
ea24c69100
commit
a9eb8e75a3
@ -1,7 +1,7 @@
|
||||
% use [nosolutions] flag to hide solutions.
|
||||
% use [solutions] flag to show solutions.
|
||||
\documentclass[
|
||||
nosolutions,
|
||||
solutions,
|
||||
singlenumbering
|
||||
]{../../resources/ormc_handout}
|
||||
|
||||
|
@ -15,17 +15,22 @@ $$
|
||||
for integer coefficients $a_i$.
|
||||
|
||||
\problem{}
|
||||
Which of the following generate $\mathbb{Z}^3$?
|
||||
Which of the following generate $\mathbb{Z}^2$?
|
||||
\begin{itemize}
|
||||
\item $\{ (1,2), (2,1) \}$
|
||||
\item $\{ (1,0), (0,2) \}$
|
||||
\item $\{ (1,1), (1,0), (0,1) \}$
|
||||
\end{itemize}
|
||||
|
||||
\begin{solution}
|
||||
Only the last.
|
||||
\end{solution}
|
||||
|
||||
\vfill
|
||||
|
||||
\problem{}
|
||||
Find a set of vectors that generates $\mathbb{Z}^2$.
|
||||
Find a set of vectors that generates $\mathbb{Z}^2$. \\
|
||||
$\{ (0, 1), (1, 0) \} doesn't count.$
|
||||
|
||||
\vfill
|
||||
|
||||
@ -40,8 +45,6 @@ Find a set of vectors that generates $\mathbb{Z}^n$.
|
||||
\definition{}
|
||||
A \textit{fundamental region} of a lattice is the parallelepiped spanned by a generating set. The exact shape of this region depends on the generating set we use.
|
||||
|
||||
\vfill
|
||||
|
||||
\problem{}
|
||||
Draw two fundamental regions of $\mathbb{Z}^2$ using two different generating sets. Verify that their volumes are the same.
|
||||
|
||||
|
@ -3,6 +3,11 @@
|
||||
\theorem{Blichfeldt's theorem}
|
||||
Let $X$ be a finite connected region. If the volume of $X$ is greater than $1$, $X$ must contain two distinct points that differ by an element of $\mathbb{Z}^n$. In other words, there exist distinct $x, y \in X$ so that $x - y \in \mathbb{Z}^n$.
|
||||
|
||||
\vfill{4mm}
|
||||
|
||||
Intuitively, this means that you can translate $X$ to cover two lattice points at the same time.
|
||||
|
||||
|
||||
\problem{}
|
||||
Draw a region in $\mathbb{R}^2$ with volume greater than 1 that contains no lattice points. Find two points in that region which differ by an integer vector.
|
||||
\hint{Area is two-dimensional volume.}
|
||||
@ -33,6 +38,10 @@ The following picture gives the idea for the proof of Blichfeldt's theorem. Expl
|
||||
\problem{}
|
||||
Let $X$ be a region $\in \mathbb{R}^2$ of volume $k$. How many integral points must $X$ contain after a translation?
|
||||
|
||||
\begin{solution}
|
||||
$\lceil k \rceil$
|
||||
\end{solution}
|
||||
|
||||
\vfill
|
||||
|
||||
\definition{}
|
||||
@ -75,7 +84,7 @@ Let $K$ be a region in $\mathbb{R}^2$ satisfying \ref{mink}. Scale this region b
|
||||
|
||||
\begin{itemize}
|
||||
\item How does the volume of $K'$ compare to $K$?
|
||||
\item Show that the sum of any two points in $K'$ lies in $K$
|
||||
\item Show that the sum of any two points in $K'$ lies in $K$ \hint{Use convexity.}
|
||||
\item Apply Blichfeldt's theorem to $K'$ to prove Minkowski's theorem in $\mathbb{R}^2$.
|
||||
\end{itemize}
|
||||
|
||||
|
@ -12,7 +12,7 @@ Show that if $r < \frac{1}{\sqrt{R^2 + 1}}$, you have at least one directon with
|
||||
\hint{Take a look at the ray through the point $(R, 1)$ and calculate the distance from the closest integer points to the ray.}
|
||||
|
||||
\begin{solution}
|
||||
Consider the ray from the origin through the point $(R, 1)$. Clearly, the two closest lattice points are $(1, 0)$ and $(R − 1, 1)$. They are equally far from the ray so let's calculate the distance from $(1, 0)$ to our ray. Call this distance $\delta$. Consider the triangle with vertices $(0, 0)$, $(1, 0)$, and $(R, 1)$. Then the area of this triangle is $12$. On the other hand, the area is also given by $\frac{1}{2} \delta \sqrt{R^2 + 1}$. So, $\delta = \frac{1}{\sqrt{R^2+1}}$. Therefore, if $r < \frac{1}{\sqrt{R^2+1}}$, we will have a clear line of sight given by this ray.
|
||||
Consider the ray from the origin through the point $(R, 1)$. Clearly, the two closest lattice points are $(1, 0)$ and $(R - 1, 1)$. They are equally far from the ray so let's calculate the distance from $(1, 0)$ to our ray. Call this distance $\delta$. Consider the triangle with vertices $(0, 0)$, $(1, 0)$, and $(R, 1)$. Then the area of this triangle is $\frac{1}{2}$. On the other hand, the area is also given by $\frac{1}{2} \delta \sqrt{R^2 + 1}$. So, $\delta = \frac{1}{\sqrt{R^2+1}}$. Therefore, if $r < \frac{1}{\sqrt{R^2+1}}$, we will have a clear line of sight given by this ray.
|
||||
\end{solution}
|
||||
|
||||
|
||||
@ -39,6 +39,12 @@ Show that there is no line of sight through the orchard if $r > \frac{1}{R}$. Yo
|
||||
Prove that there exists a rational approximation of $\sqrt{3}$ within $10^{-3}$ with denominator at most $501$. Come up with an upper bound for the smallest denominator of a $\epsilon$-close rational approximation of any irrational number $\alpha > 0$. Your bound can have some dependence on $\alpha$ and should get smaller as $\alpha$ gets larger. \\
|
||||
\hint{Use the orchard.}
|
||||
|
||||
\begin{solution}
|
||||
Take the line through the origin of slope $\sqrt{3}$. We would like an orchard for which $r = 10^{-3}$ gives no line of sight, since this will guarantee an integer point within a distance of $10^{-3}$. Then by our previous problem, we can take $10^{-3} > \frac{1}{R}$, so take $R > 1000$. Now since this line intersects the boundary of the orchard at $( \frac{R}{2}, \frac{\sqrt{3R}}{2} )$, we have that the $x$-coordinate is at most $\frac{R}{2} = 501$. Then we have that our lattice point $(x, y)$ satisfies $\sqrt{3x} - 10^{-3} < y < \sqrt{3x} + 10^{-3}$, so $\sqrt{3} - 10^{-3} < \frac{x}{y} < \sqrt{3} + \frac{10^{-3}}{x}$. Therefore, $\frac{y}{x}$ is a rational approximation that is $10^{-3}$-close to $\sqrt{3}$ and has denominator at most $501$. Notice that we got closer than we need to.
|
||||
|
||||
Repeating this same process, our upper bound for the denominator of an $\epsilon$-close approximation of $\alpha$ is $\frac{\text{cos}({\text{atan}({\alpha})})}{\epsilon} = \frac{1}{\sqrt{\alpha^2 + 1 \epsilon}}$.
|
||||
\end{solution}
|
||||
|
||||
\vfill
|
||||
\pagebreak
|
||||
|
||||
|
Loading…
x
Reference in New Issue
Block a user